LSAT and Law School Admissions Forum

Get expert LSAT preparation and law school admissions advice from PowerScore Test Preparation.

User avatar
 Dave Killoran
PowerScore Staff
  • PowerScore Staff
  • Posts: 5852
  • Joined: Mar 25, 2011
|
#43442
Complete Question Explanation
(The complete setup for this game can be found here: lsat/viewtopic.php?t=16406)

The correct answer choice is (B)

Answer choice (B) is correct because F and H in group 1 automatically bring along J (from the first rule) and L (from the third rule). However, that means FJHL completely occupy group 1, forcing G and M into group 2 together, a violation of the second rule.

Note that the hypothetical produced in question #1 eliminates answer choices (D) and (E), and the hypothetical produced in question #4 eliminates answer choice (A). The hypothetical produced in question #2 also helps in eliminating answer choice (C).

Get the most out of your LSAT Prep Plus subscription.

Analyze and track your performance with our Testing and Analytics Package.